QOD

अब Quizwiz के साथ अपने होमवर्क और परीक्षाओं को एस करें!

A practice is using an FeLV test with a sensitivity of 90% and a specificity of 95%. Assuming the prevalence of feline leukemia in the area is 5%, what is the predictive value positive (PVP) of the test? A - 45% B - 48% C - 55% D - 88% E - 90%

It is 48%. The trick with this kind of question is to pick an imaginary number of animals that you test, like 1000, and fill out your 2x2 table from there. Follow the links to see diagrams step by step. If prev is 5% then there must be 50/1000 cats with FeLV and 950 cats that are disease-free. A 90% sensitive test will correctly call 45/50 positive (box "a"), and IN-correctly call 5/50 negative, (box "c": these are the false negs). If 50/1000 animals are infected, then 950/1000 are disease-free. Your 95% specific test will correctly call 902/950 disease-free (box "d": 0.95 X 950=902) and IN-correctly call 48/950 positive, (box "b": these are the false pos). Now your a,b,c,d boxes are all filled, it is easy to calculate PVP =a/(a+b)=45/(45+48)=48%

Which one of the following represent the most effective control measures in herds infected with porcine reproductive and respiratory syndrome (PRRS) virus? A - Cull all sows after first farrowing B - Maintain therapeutic levels of streptomycin in feed C - Nursery depopulation, segregated early weaning 12-18 mos after outbreak D - Buy PRRS-negative gilts, test on arrival, segregate 45 days, retest before join herd E - Vaccinate boars in contact with sows, maximize pen ventilation

12-18 months after the initial outbreak, sows are usually no longer transmitting the Porcine reproductive and respiratory syndrome (PRRS) virus, so segregated early weaning and removal of nursery pigs (with concurrent disinfection of nursery before repopulation) may help control the virus. (C) Because it appears that sows generally only abort once, culling after the first farrowing not advisable. Antibiotic administration can help control secondary bacterial infections, but not the arterivirus responsible for PRRS. Adding negative gilts to a herd already infected with the virus is unlikely to result in control.

You must treat a 1000 lb cow infected with Eimeria bovis (an intestinal parasite) using 9.6% amprolium oral solution. The dose is 10mg/kg PO SID for 5 days. How much amprolium do you give this cow, and how often each day? A - 4.7 cc, once a day B - 96 cc, twice a day C - 47 ml, once a day D - 9.6 ml, once a day

47 ml, once a day. A cc is cubic centimeter, which is the same volume as a milliliter, (ml). SID means once a day. 9.6% concentration contains 96 mg amprolium per ml solution. 1000 lb cow = 454 kg, (1000lb / 2.2lb/kg=454 kg). The dose for this cow will be 454 kg x10 mg/kg =4540 mg. To get the dose in ml divide 4540 mg by 96 mg/ml 4540 mg / 96mg/ml =47ml Eimeria (and isospora) are intestinal protozoal parasites (also called coccidia).

Which three nerve blocks will correctly isolate the majority of equine lamenesses? A - Heel, High 4-point, Proximal metacarpal B - Suspensory ligament infiltration, Coffin, Proximal metacarpal C - Low 4-point, Fetlock, Peroneal D - Median/Ulnar, Low volar, Navicular bursa E - Palmar digital, Abaxial sesamoid, Low palmar

90% of all lameness is in the HOOF. That means the three LOWEST nerve block will help you isolate most lamenesses. These 3 are: 1). HEEL block (also called palmar digital (PD), used to help Dx caudal heel pain (navicular syndrome) 2). PASTERN block (also known as the foot block, abaxial sesamoid nerve block (ASNB))- Can use to make a horse with acute laminitis comfortable) 3). FETLOCK block (Also called Low palmar, Volar or Low 4-point block). If horse is still lame after these 3 blocks (heel, pastern, fetlock), continue up limb with metacarpal/high 4-point, then prox. metacarpal blocks next.

A 5-month old English sheepdog with unilateral peri-ocular alopecia is presented for a routine vaccination visit. A skin scrape shows the following. http://www.merckmanuals.com/vet/zk/multimedia/v4737616/n/x.html What is the treatment plan? A - Do nothing B - Weekly Amitraz dip C - Ivermectin 0.3-0.6 mg/kg PO SID 30-60 days D - Imipramine 2 mg/kg PO SID 30-60 days E - Topical imidacloprid and moxidectin, monthly

90% of localized Demodex cases resolve without treatment. 10% go on to generalized demodicosis. Adult-onset demodicosis is often severe and refractory to treatment. Need to advise owner that many adult-onset cases are medically controlled, but not cured. Avoid Ivermectin in Collies, Shetland and Eng. Sheepdogs, herding breeds or mixed breeds with these breeds in them.

An alpaca is presented in poor body condition with an unkempt coat, patchy alopecia and severe pruritis and crusting of the extremities. What is the most appropriate diagnostic step? A - Tell Yoda you found his brother B - Test skin crusts for contagious ecthyma parapoxviruses C - Check thyroid status and bluetongue titers D - Punch biopsy E - Skin scraping and microscopic exam

? Path we said: Unkempt coat: parasitic, metabolic, allergens, autoimmune, weather, husbandry Patchy alopecia - parasites, fungus, bacteria, endocrine/metabolic Pruritis - parasites, bacterial, fungal, allergies, neurogenic, metabolic Crusting of extremities - see above list, extremities! Important. Parapoxvirus - crusting around the mouth, teat and distal extremity, not itchy Bluetongue- virus spread by insects, lesions on feet and a blue tongue Punch biopsy and skin scraping - both work, but skin scraping is less infective

What is a gilt? A - Castrated male sheep B - Female pig that has not had a litter yet C - A male pig, castrated before puberty D - Ewe that has lambed at least once E - Female pig that has had more than one litter

A young female pig that has not had a litter yet is called a gilt A male pig, castrated before puberty is called a barrow A wether is a castrated male sheep. FYI-A female ferret is called a Jill and a male is called a Hob

What side effect is often seen in horses tranquilized with acepromazine? A - Hives B - Penis protrusion C - Collapse D - Vomiting

Acepromazine can cause penis protrusion in large animals, especially horses. Remember that GIANT breeds and SIGHT hounds can be very sensitive to acepromazine. Acepromazine is sometimes used as an anti-emetic (ANTI-vomiting) drug

How long after the first rabies vaccination is a dog, cat or ferret considered to be immunized and protected against rabies? A - Same day B - After 24 hours C - After 7 days D - After 14 days E - After 28 days

After 28 days. According to the Compendium for Rabies Control, a peak rabies virus antibody titer is reached 28 days after initial vaccination and immediately after booster vaccination. Here are some thoughts on rabies: When in doubt, it is never wrong to check with your local health department. Basically all potential rabies exposures boil down to 2 questions: 1. Who is involved? Animal-Animal exposure (less alarm bells) Animal bites/exposes human (more alarm bells) 2. Was animal vaccinated/up to date on vaccination? Up to date on vaccs (less alarm bells, shorter observation) Vaccinated, but not up to date (Handle on case-by-case basis) Un-vaccinated pet (more alarm bells, euthanize or long observation period) Wild animal, esp. bats, raccoon, skunk (euthanize, send head to state lab) When dealing with rabies questions, ask yourself if this seems like a HIGH-risk exposure (ie: wild raccoon bites a child) or a LOWER risk exposure (ie: Up-to-date vaccinated dog messes with woodchuck but no bite wounds on dog). For high risk lean towards euthanasia/testing or long quarantine. For low risk lean towards short observation period (10 days) and a rabies booster.

A 2 year-old Hereford cow is presented with a 1-week history of blepharospasm, lacrimation and corneal opacity. There is a large central corneal abscess secondary to suspected infectious keratoconjunctivitis (pinkeye). What is an appropriate treatment plan? eye A - Subconjunctival oxytetracycline, Topical polymyxin B ointment B - Systemic oxytetracyline, Subconjunctival penicillin C - Systemic long-acting penicillin-gentamicin D - Systemic Florfenicol, Topical erythromycin ointment E - Systemic Sulfamethazine, Topical atropine ointment

Ampicillin, penicillin and kanamycin can be injected subconjunctivally; best results are obtained with injection into the bulbar conjunctiva. In the past, veterinarians used gentamicin subconjunctivally, but this is now a gray area of off-label use, because gentamicin is not FDA-approved for systemic use in cattle. Oxytetracycline is generally considered the drug of choice for systemic therapy because it is concentrated in corneal tissue. Oxytetracycline cannot be injected in the subconjunctiva because it will cause conjunctival necrosis. Two injections (20 mg/kg, IM) of a long-acting oxytetracycline formulation (200 mg/mL) at 72-hr intervals is the systemic treatment of choice.

A tourniquet was placed at mid cannon bone on this cow's leg. Lidocaine is being injected in the dorsal pedal vein. What is expected to happen? [image] A - Anesthesia distal to tourniquet B - Bradycardia with gradual tourniquet removal C - Anesthesia only from mid-pastern to the toe D - Cessation of atrial fibrillation after tourniquet removal E - Anesthesia only of skin in the region of the injection

Anesthesia distal to tourniquet. This is an example of intravenous regional anesthesia, the anesthetic method of choice for most bovine digital surgical procedures. It has similar results to a nerve block, but is easier to do with reliable results. You need only one injection and no special knowledge of anatomy. 2% Lidocaine infuses down the venous system and anesthetizes the entire lower limb distal to the tourniquet within about 10 minutes. The amount of lidocaine needed is typically 10-30 ml and should not exceed a total dose of 9 mg/kg. Click here to see tourniquet sites and veins for intravenous regional anesthesia of the distal bovine hindlimb. Do not keep a tourniquet in place more than one hour. When surgery is complete, slowly release the tourniquet and then retighten it. If antibiotics are indicated, infuse them now then release the tourniquet after a few more minutes. Some references call this a "Bier" block, after the pioneering German physician August Bier, who reported on this form of anesthesia in 1908. Lidocaine (and procainamide) are used to treat arrhythmias, but more for ventricular fibrillation in small animals and not administered from the foot. Think of quinidine sulfate to treat atrial fibrillation in symptomatic horses.

If the following image is seen on necropsy, what would be recommended as a treatment plan for the remaining group of pigs? image: http://www.merckmanuals.com/vet/zk/multimedia/v4732415/n/x.html?qt=&sc=&alt= A - There is no effective treatment B - Clorsulon PO, all animals C - Lincomycin IM, underweight animals D - Decrease non-protein nitrogen in diet E - Antihelminthics

Antihelminthics are the treatment of choice. This is ascarid (roundworm) infestation and these are classic "milk spots"- liver scars left by migrating ascarid larva traveling to the lungs. In heavy infestation, larvae can cause pulmonary edema, consolidation, severe respiratory distress. May see icterus. Follow this link to see another image of milk spots. A fecal exam would show eggs that look like this. Rx with antihelminthics, like benzimidazoles, ivermectin, pyrantel, levamisole. May need antibiotics to treat secondary bacterial pneumonia, but primary Rx is antihelminthics. Lincomycin is a lincosamide antibiotic used in pigs against mycoplasma pneumonia. Clorsulon is used to treat liver flukes in cattle and sheep. Click here to see an adult ruminant liver fluke, Fasciola hepatica. Click here to see a fluke egg.

During a routine immunization visit for a 2 year-old neutered male Newfoundland dog, a systolic ejection-type (crescendo-decrescendo) murmur is detected, audible loudest on the left side of the chest between the 2nd and 5th intercostal (IC) space and at the thoracic inlet lateral to the trachea. Which condition is highest on a differential diagnosis list? A - Pulmonic stenosis B - Mitral dysplasia C - Tricuspid dysplasia D - Aortic stenosis E - Patent ductus arteriosus (PDA)

Aortic stenosis (also called sub-aortic stenosis [SAS]), is a systolic, ejection-type (crescendo-decrescendo) heart murmur which may be heard most loudly on the left chest between the 2nd and 5th intercostal (IC) space or at the thoracic inlet (lateral to trachea). Inherited in Newfoundlands. Predilection in many BIG BREEDS- German Shepherd, Golden Retriever, Boxer, Rottweiler. Mitral dysplasia and other mitral valve problems are heard further back on left at 5th-6th IC. More common in CATS. With pulmonic stenosis see RIGHT ventricular hypertrophy, because pulmonic valves blocks outflow from R ventricle (mostly dogs). Follow this link to see a Merck image of Rt. ventricular hypertrophy. Tricuspid dysplasia is heard further back on RIGHT at 5th-6th IC. Uncommon. Expect a continuous murmur with Patent ductus arteriosus (PDA). Vast majority detected at first vaccination visit.

What is the best way to evaluate the body condition of an adult llama? A - Palpate over the lumbar vertebrae B - Feel the intercostal spaces C - Digital exam of the supraorbital fossa D - Rectal exam to evaluate pelvic fat E - Weight X height at withers X 0.28

Assess body condition of llamas and alpacas by palpating the amount of tissue over the lumbar vertebrae. South American camelids are adapted to high mountain environments and diets. Most adult males (and females through midgestation), maintain appropriate body condition on 10-14% crude protein grass hay with total digestible nutrients (TDN) of 50-60%. Late gestation and heavily lactating females need a higher percentage of crude protein and TDN of 65-70%. Excess legumes in the diet are not typically necessary and may cause obesity.

During the fall, a 5 year-old standardbred mare from northern California is presented with a 5-day history of depression, partial anorexia and undulating fever between 102-106 F (38.9-41.1 C)..[N=99-101.3 F]. Physical exam reveals mucosal petechiae and icterus. The horse has edematous hindlimbs and appears reluctant to move. Blood drawn for a complete blood count shows inclusion bodies in the neutrophils. What is the diagnosis? A - Equine Babesiosis B - Equine ehrlichiosis C - Equine viral arteritis (EVA) D - Purpura hemorrhagica E - Equine infectious anemia (EIA)

B This is one of the 5 classic "anemia / edema " presentations of horses. (Remember "Big 3 are PEE" Purpura, EIA, EVA; 2 minors are Babesia, ehrlichia (now renamed Anaplasma). Inclusion bodies in the neutrophils of a California horse with icterus and petechiae and an undulating fever says Equine granulocytic ehrlichiosis (EGE). Originally classified as Ehrlichia equi, but is NOW called ANAPLASMA phagocytophila due to DNA sequencing studies. EGE is a seasonal necrotizing vasculitis (edema, icterus, petechiae) seen in N. California, suspected to be tick borne. See dependent edema with Equine viral arteritis (EVA), but also should see respiratory presentation (conjunctivitis/pinkeye, rhinitis) +/- abortions. Equine infectious anemia (EIA) is rarely seen today because of testing programs, but could present this way. Less likely because 92% cases seen around the Gulf of Mexico states (Texas to Florida), Mississippi river valley AND no inclusion bodies. Purpura hemorrhagica is a type III Antigen/Antibody/Complement complex disease causing vascultis, petechia, purplish discoloration, usually afebrile, with Hx of recent Strangles (Strep equi equi) or bacterin vaccination. Equine Babesiosis presents more as a hemolytic anemia: seen in the S. USA, endemic in Southern FLORIDA

Which forceps secure drapes to the patient by penetrating the skin? A - Backhaus B - Roeder C - Brown-Adson D - Crile

Backhaus and Roeder forceps are towel clamps used to secure drapes to the patient's skin. Backhaus forceps look like the sharp pincers of an ant, and penetrate both the cloth drapes and the patient's skin. Roeder forceps look like Backhaus clamps, but they have a metal ball welded to the points of the jaws to prevent tissue penetration.

Where should a feline leukemia (FeLV) vaccine be injected? A - Cervical interscapular region B - Below stifle, lateral left hind C - Above stifle, lateral right hind D - Above stifle, lateral left hind E - Below stifle, lateral right hind

Below stifle, lateral left hind. Feline leukemia virus (FeLV) and rabies vaccinations have been associated with sarcomas. You should always keep a record of where vaccinations were given. Remember your L's and R's for feline vaccination: For FeLV, vaccinate LOW and LATERAL on the LEFT hind. For Rabies vaccinate low and lateral on the RIGHT hind. Here is a direct quote from the AAFP Feline Vaccine Advisory Panel Report (full report- see Appendix 2, p. 1440) "FeLV or FIV antigen (plus any other antigen except rabies) should be administered subcutaneously (SC) on the lateral side of the left hind limb below the stifle joint (vaccine-associated sarcomas arising in the proximal femoral area are difficult to completely excise; placement of vaccines in this area is strongly discouraged)." Click here for a summary of feline vaccine guidelines

An 18 month old steer is presented that is just down from summer pasture in the mountains of Colorado with brisket edema, slight dyspnea and distended jugular veins. What condition is at the top of the differential diagnosis list? A - Hardware disease B - Cardiac lymphosarcoma C - White muscle disease D - High Mountain disease E - Cor pulmonale

Bilateral dilated jugulars says "Heart problem". Hx of high altitude exposure and brisket / submandibular edema point towards High Mountain disease. Remember Locoweed (Astragalus and Oxytropis) ingestion worsens disease, due to swainsonine.

Which reportable condition can affect cattle, but mainly causes disease in sheep? A - Anthrax B - Rinderpest C - Vesicular stomatitis D - Malignant Catarrhal fever E - Bluetongue

Bluetongue is almost exclusively a sheep disease, (but cattle and deer can get it). Rinderpest mainly affects cattle and is reported to be eliminated as of October 2010 by the United Nation's Global Rinderpest Eradication Program. Because it is a classic severe and reportable vesicular disease, vets will likely need to keep rinderpest on their mental DDX list for years to come. Pseudorabies is basically a pig pathogen. Can affect cows, but horses (and humans) are resistant Vesicular Stomatitis (VS) can occur in horses, pigs, cows. Remember the big 8 vesicular diseases: BVD, IBR, BPS, MCF, Bluetongue, VS, FMD, Rinderpest) Refs: Pasquini's Guide to Bovine Clinics, 4th ed. pp 8-11, 247, Pasquini's Guide to Equine Clinics, 3rd ed. pp. 289, 302 and the Merck Veterinary Manual online edition.

Which parasite has been associated with a hypoadrenocorticism-like syndrome and has been suggested as a cause for cecocolic intussusception in dogs? A - Ancylostoma caninum B - Physaloptera spp C - Trichuris vulpis D - Toxocara canis E - Spirocerca lupi

C Whipworms, (Trichuris spp) are typically found in the cecum and large intestine. Mainly in dogs, rare in cats. Trichuris suis in pigs can cause unthriftiness in younger animals. If clinical, look for signs of large bowel diarrhea (frequent urgent defecation of loose watery feces, possibly with mucus or fresh blood). Can be associated with a hypoadrenocorticism-like syndrome (hyponatremia, hyperkalemia, azotemia, metabolic acidosis). Whipworm infection has been suggested as one cause of cecocolic intussusception. Physaloptera spp (Stomach worms) may cause vomiting, anorexia, dark feces. Spirocerca lupi makes nodules in the esophageal, gastric, or aortic walls. Typically asymptomatic. Roundworms (Toxocara canis) may cause visceral and ocular larva migrans. Hookworms (Ancylostoma spp) may cause cutaneous larva migrans.

Which three diseases are on the differential diagnosis list when encountering sudden death in a young, fast-growing calf? A - Lasalocid toxicity, Salmonellosis, Lymphosarcoma-juvenile form B - Salmonellosis, Colibacillosis, Enzootic Calf Pneumonia C - White muscle disease, Enterotoxemia, Colibacillosis D - Bovine Viral Diarrhea, Lasalocid toxicity, White muscle disease E - Winter dysentery, Enterotoxemia, IBR-encephalitic form

C- White muscle disease, Enterotoxemia, Colibacillosis When you hear "Sudden Death" in calves, think White Muscle Disease caused by Selenium/vitamin E deficiency; Think Enterotoxemia caused by Clostridium perfringens type B or C. And think Severe Colibacillosis. Can also see sudden death with encephalitic form of IBR and severe Salmonellosis.

A 3-day old female alpaca (cria) is presented in respiratory distress. The cria's cheeks flare noticeably during inspiration, and the distress is more pronounced during nursing, when the animal gasps and inhales milk. What is the most likely diagnosis? A - Choanal atresia B - Wry face C - Diaphragmatic hernia D - Lung lob torsion E - Mitral stenosis

Choanal atresia is one of the most common congenital problems of South American camelids. Choanal atresia occurs when the caudal nares (choanae) fail to open during embryologic development. Can be unilateral or bilateral and may cause complete or partial blockage.

Which of the following hays is highest in calcium? A - Alfalfa hay B - Barley hay C - Bermuda grass hay D - Orchard grass hay E - Oat hay

Compared to other types of hays, alfalfa tends to be higher in calcium, protein, and energy. Click here to see more on pasturing horses and also equine parasite control. Click here to see Common Nutritional Feed Profiles, courtesy of Equi-Analytical Laboratories.

Which pair of neonatal calf diarrheas both have public health/zoonotic significance? A - Colibacillosis, Rotavirus B - Salmonella, Coccidiosis C - Clostridium perfringens, Coronavirus D - Cryptosporidiosis, Salmonella E - Colibacillosis, Ostertagiasis

Cryptosporidiosis is caused by a protozoa implicated in drinking water-associated outbreaks of diarrhea in humans; Salmonellae can infect humans from a number of different sources (turtles, chickens, eggs) E. Coli can cause human disease, but Ostertagia and bovine rotavirus do not. (There IS a human version of rotavirus, however) C. perfringens can be found in soil and in the normal gut flora and is not contagious.

A 5-year old Holstein cow is presented with a 2-day history of being off feed and a precipitous drop in milk production. T=103 F (39.4 C)..[N=101.5-103.5 F] HR=132 bpm........[N=55-80] RR=36 brpm.........[N=10-30] The cow stands with abducted elbows, an arched back and is reluctant to move. Physical exam shows intermandibular edema and bilateral jugular distention. A grunt is heard when pressure is applied to her xiphoid and there is a washing machine murmur (almost like splashing sounds) on both sides. There is little rumen activity. What is the recommendation for the farmer? A - Treat with high-dose penicillin/streptomycin B - Check the feed for excess monensin/lasalocid C - Test the herd for bovine leukosis D - Check the feed for cottonseed meal (gossypol) E - Cull this cow

Cull this cow. Bilateral dilated jugulars say "Severe heart problem". A washing machine murmur and the painful stance, as well as respiratory grunting all point to hardware disease (Traumatic reticuloperitonitis). Treatment is unrewarding- CULL. Prevent problem by making every cow swallow a small bar magnet to attract and hold nails, wire, sharp metal inside the reticulum. COOL FACT: A compass can tell you if a cow has a magnet. Hold it near the brisket. If there is a magnet, compass needle will point to the cow, even if you move the compass. Cardiac lymphosarcoma may present with heart failure signs, but less likely to have such an acute onset of agalactia, painful stance or classic "washing machine" murmur. Monensin/Lasalocid are ionophore coccidiostats associated with cardiac failure- MOST TOXIC to HORSES Cottonseed meal contains Gossypol - see cardiac toxicity/dyspnea/ sudden death in calves; sterility/decreased conception in adults. are associated with cardiac toxicity.

A client is considering the purchase of a 12-year old prize-winning thoroughbred mare for breeding. The pathology report from an endometrial biopsy says the mare is category III by the Kenney scoring system. What advice is most appropriate to give the client? A - Don't buy this mare for breeding B - 10%-50% chance of bringing a foal to term C - 80%-90% chance of bringing a foal to term D - 50%-80 chance of bringing a foal to term E - Best to breed by artificial insemination

Don't purchase a category III mare for breeding. Endometrial biopsy predicts a mare's ability to carry a foal to term and category III is the worst news. (Remember "3 strikes and you are out"). Endometrial biopsies are classified into 3 types by the Kenney system according to degree of periglandular uterine fibrosis and scarring. The Kenney system gives a % probability that a mare can carry a foal to term. Category I is best. Most horses fall in category II. Category III is worst. Category I No changes (80%-90%) Category IIA Minor changes (50%-80%) Category IIB Moderate changes (10%-50%) Category III Severe changes. Less than 10% probability can carry a foal to term.

An outbreak of diarrheal disease of piglets has occurred which affected the healthiest animals in the herd, 1-2 weeks after weaning. Some affected piglets had no signs except peracute death. Other affected piglets exhibit diarrhea, ataxia, paralysis, and recumbency. What condition is at the top of the differential diagnosis list? A - Clostridium perfringens type C enteritis B - Edema disease C - Porcine proliferative enteritis D - Epidemic transmissible gastroenteritis (TGE) E - Hemagglutinating encephalomyelitis virus (HEV)

Edema disease is caused by entertoxigenic E. coli (ETEC). Look for severe acute illness ranging from peracute death with no signs to CNS involvement with ataxia, paralysis, and recumbency in healthiest pigs 1-2 weeks after weaning. Hemagglutinating encephalomyelitis virus (HEV), is almost exclusive to piglets less than 4 weeks old. Two clinical presentations: Vomiting and wasting disease (VWD) and encephalitic. Clostridium perfringens type C enteritis ,also called enterotoxemia in other species is characterized by a HEMORRHAGIC diarrhea in 1-3 day old piglets. Porcine proliferative enteritis is principally a diarrheal disease of growing finishing (40- to 80-lb) pigs and young breeding pigs. Epidemic transmissible gastroenteritis (TGE) in non-immune pig herds characterized by HIGH MORBIDITY and HIGH MORTALITY in piglets less than 1 week old.

After diagnosis of appendicular osteosarcoma in a dog, the leg is amputated and chemotherapy is initiated. At follow-up monitoring exam 3 months later, thoracic radiographs are clear. What finding would suggest a guarded prognosis? A - High alkaline phosphatase B - Depressed calcium-phosphorus ratio C - Microcytosis D - Polycythemia E - Elevated albumin, depressed globulin

Elevated alkaline phosphatase is associated with a poor prognosis prior to surgery for osteosarcoma. If alkaline phosphatase stays elevated postoperatively, the prognosis is even more guarded. The median survival of dogs with appendicular osteosarcoma is only 4-5 months after amputation alone, and 10-12 months with amputation plus chemotherapy. 90-95% of animals with osteosarcoma ultimately develop visible metastatic disease. The foundation of monitoring for osteosarcoma after amputation and adjunct chemotherapy is 3-view thoracic radiographs every 2-3 months. Click here to see a radiograph, metastatic lung tumors in a dog, and another radiograph of feline metastatic lung tumors. Think of microcytotic anemia with iron deficiency in young rapidly-growing animals like piglets and puppies and with portosystemic shunts. Think of polycythemia (high RBCS) most commonly secondary to dehydration.

Most of a litter of European wild boar at a zoological park have died. The surviving piglets are in lateral recumbency with a frothy nasal discharge. Necropsy of the piglets reveals pulmonary edema and copious fluid in the trachea and bronchi along with grayish- white necrotic foci on the myocardium. Which one of the following diseases is the most likely diagnosis? A - Edema disease B - Encephalomyocarditis C - Glasser's Disease D - Porcine Respiratory and Reproductive Syndrome E - Pseudorabies

Encephalomyocarditis virus (EMCV) is caused by a cardiovirus in the family picornaviridae. Confusingly, the virus is named for its effects on mice. Think of rodents and exotic zoo mammals with EMCV. Pig-to-pig contact, contamination of swine feed and water by rodents or ingestion of dead rodents may cause disease. See pulmonary edema and copious transudate in the respiratory tract, causing cardiac failure. Zoo outbreaks of EMCV have included lions, African elephants, rhinos, hippos, sloths, llamas, antelope and nonhuman primates. An outbreak of lion deaths at a Florida zoo in the USA occurred after feeding them the carcass of an African elephant that had died of EMCV. Edema disease is a neurologic disease caused by a hemolytic Escherichia coli producing Shiga toxin e2 and F18 pili resulting in high mortality in recently-weaned pigs. Glasser's Disease, caused by Hemophilus parasuis is usually an acute disease of 6 to 8 week-old pigs which causes fibrinous arthritis, polyserositis, and meningitis. Porcine Reproductive and Respiratory Syndrome (PRRS), is an arterivirus causing reproductive failure and post-weaning respiratory disease. Pseudorabies is a herpesvirus: infection causes CNS disease in neonates, respiratory disease in weaned pigs, and fever in all ages.

What lab result would you expect to find in a cat with Diabetes mellitus? A - Increased urine glucose; Decreased blood glucose B - Low urine glucose; Normal blood glucose C - Increased urine glucose, Increased blood glucose D - Urinary infection; Low blood glucose

Expect an animal with diabetes mellitus to have both a HIGH urine glucose and also a HIGH blood glucose. Typically do fast urine dipstick and blood dipstick tests first, as well as check the urine specific gravity when diabetes is suspected. Diabetes mellitus is a complicated disease with a complicated treatment, that requires the owner to inject insulin and monitor urine glucose each day. A technician's job usually involves helping to educate the owner, or in the nursing care of diabetics with a severe form called diabetic ketoacidosis (DKA).

A potbellied pig is presented acutely lame and non-weight-bearing on the right fore after jumping off of a sofa. A radiograph of the affected limb is shown below. What is the most accurate interpretation? http://www.merckmanuals.com/vet/zk/multimedia/v4735319/n/x.html A - Dislocation of distal radius and ulna B - Salter III fracture, olecranon C - Fracture, greater tubercle and humeral head D - Probable triceps brachii injury, no obvious skeletal damage E - Distal humeral condyle fracture

Fractures of the distal humerus are the most common fracture types seen in potbellied pigs, usually as a result of jumping off furniture. Other fracture sites are related to dog bites (elbow), or restraint (elbow, femur) and horse kicks (femur). Click here to see a radiograph of a normal humerus in a potbellied pig and compare it to this radiograph of a distal humeral fracture at the humeral condyle. Bonus: Here is a normal abdominal-pelvic radiograph of a young potbellied pig. Note the relative small stature of the bones in relation to the heavy body and abdominal contents, so thick they hardly allow x-ray penetration. These are heavy, hard-to-handle animals and it is easy to imagine the stress of even a short fall breaking bones.

A pig farmer complains of strange behavior in his feeder pigs. Most are hyperexcitable yet are not squealing. A few are lethargic, wandering aimlessly and seem to be blind. Bloodwork shows a marked increase in sodium concentration. Upon inspection of the pen, it turns out that the waterer had been inadvertently turned off. Which one of the following treatments is indicated? A - Frequent small amounts of water B - Ad lib water C - Mannitol D - Furosemide E - Ad lib water and IV 1/2 strength Ringers solution

Frequent small amounts of water. It is dangerous to rapidly correct for a hyperosmolar or hypoosmolar state (salt toxicity) and could lead to brain swelling. It is best to gradually rehydrate the animals with small amounts of water given frequently. Also, IV 1/2 strength Ringers solution with 2-1/2 % dextrose can be given at a slow rate.

Which statement about the proper storage of fresh or frozen plasma is most correct? A - can be frozen for up to 1 year B - must be used within 48 h C - should be stored above 98.6 F (37 C) D - can be stored at 39.2 F - 50 F (4 C - 10 C) for up to 3 weeks

Fresh or frozen plasma is stored at -4 F to -22 F (-20 C to -30 C) for up to 1 year. Frozen plasma should be thawed to 98.6 F (37 C) in a warm water bath to 30-60 minutes before administration. Typically, plasma is given for volume expansion to animals in shock, or who had suffered heavy burns. Whole blood with acid citrate can last up to 14 to 21 days when refrigerated. Heparinized blood must be used within 48 h. Typically we give blood transfusions to animals who have suffered severe blood loss from injury or anemia. Follow this link for more on blood transfusion in animals.

Which choice lists these species in order of gestation length, from longest to shortest pregnancy? A - Cow, Horse, Pig, Dog, Goat B - Cow, Horse, Pig, Goat, Dog C - Llama, Pig, Sheep, Ferret, Cat D - Horse, Cow, Goat, Pig, Dog

Here is a list of gestation length for the major species, in order: Llama 1 year (350d), Horse 11 mo (330d), Cow 9 mo (~280 d, same as PEOPLE), Sheep/goats 5 mo (150d), Pig 4 mo (114d), Dog/ Cat 2 mo (60d), Ferret 1.5 mo (42d.). Basic questions on estrus cycle and gestation come up over and over, so commit them to memory for the major species at LEAST (Cow, Horse, Dog, Cat, Sheep/Goat, Pig). Merck has two excellent tables: In general, the BIGGER the beast, the LONGER the gestation ie: Mice are about 20 days, elephants are TWO YEARS (660 days)!

Which one of the following choices is the best post-mortem diagnostic test for scrapie in sheep? A - Histopathology of the obex B - IFA test on whole blood C - ELISA antibody test on serum D - Immunohistochemistry of the third eyelid E - Skin biopsy of scraped area

Histopathology of the obex is the post-mortem test of choice for scrapie. Immunohistochemistry of the nictitating membrane, though NOT 100% sensitive, can detect scrapie in live animals. A rectal mucosa biopsy test was approved by USDA APHIS in 2008 which is less time-consuming to perform and can be repeated. Immuno-blot testing of biopsied tonsils may detect prion proteins in subclinically-infected sheep less than 1 year old. Antibodies against scrapie/prions are NOT produced. When you think of Scrapie, remember also Bovine spongiform encephalopathy (BSE;cows) and Chronic Wasting disease (CWD; deer, elk). All of these are REPORTABLE.

A 4-year old quarterhorse mare is presented with a runny left eye and a urine-scalded perineum. No other horses on the farm are sick. Physical exam reveals a corneal ulcer and keratitis OS (left eye), and atrophy of the temporal and masseter muscles. There is decreased perineal sensation, a weak tail and weak anal sphincter with retained manure. The horse is bright, alert and responsive. T=102.2 F (39.1 C)..[N=99.0-101.3 F] HR=40 bpm............[N=28-40] RR=12 brpm...........[N=10-14] Which one of the following choices is the most likely diagnosis? A - Cauda equina neuritis B - Equine degenerative myelopathy (EDM) C - Botulism D - Nigropallidal encephalomalacia E - Equine protozoal myelopathy (EPM)

Horses with cauda equina neuritis (also called polyneuritis equi) have a progressive symetric LMN paresis of the tail, bladder, rectum, anal sphincter. Look for urinary incontinence, fecal retention and a weak or paralyzed tail. May see hind limb paresis if lumbosacral spinal cord is affected. Cranial nerves can also be affected, but typically cranial involvement is asymetric. May see temporal or masseter atrophy (Cranial Nerve 5), facial paralysis and exposure keratitis (Cranial nerve 7), head tilt or other CNS signs. Cause is unknown, may be an autoimmune process. Grave prognosis. Eventually euthanized. Herpesvirus myeloencephalopathy (EHV-1) may also present with urinary incontinence, but this is an uncommon manifestation of equine rhinopneumonitis. You would expect to hear a history of the more common EHV signs in other horses from the same farm, like respiratory disease ("snots") in foals and abortions in mares.

How long should you wait before you can definitively say a dog is cryptorchid? (Has an undescended testicle) A - Birth-2 months B - 2-6 months C - 7-12 months D - Cannot say

If the testicle(s) are not in the scrotum by 2-6 months, then the animal is cryptorchid. Normally testes descend by 6-8 wks in dogs, prenatally in cats). Cryptorchidism is COMMON (1-2% dogs, right side 75%; In horse more common on LEFT) Remember that cryptorchid dogs are 10X to 14X more likely to develop sertoli cell tumors and seminomas. Also see increased risk of testicular torsion. The treatment is CASTRATION. Discourage breeding.

The udder of a goat is cold and bluish, with pitting edema and serous bloody discharge. The animal is down and shocky and appears to have gangrenous mastitis. Which organism is usually associated with cases of severe clinical mastitis in goats and sheep? A - Streptococcus agalactiae B - Escherichia coli C - Pseudomonas aeruginosa D - Staphylococcus aureus E - Arcanobacterium (Actinomyces) pyogenes

In goats and sheep, Staphylococcus aureus can produce a necrotizing alpha toxin which can cause a gangrenous mastitis, sometimes called "blue bag". Staph. aureus is also the most common cause of regular clinical mastitis in small ruminants.(11%-63% of cases). For more on goat/sheep mastitis, see the Maryland Coop Extension website. If this were a cow down with a septic mastitis, you would think of coliform mastitis (E. coli).

Several sheep aged 1-2 years in a herd of 300 were observed lagging and acting depressed in late summer. This morning, two animals found in sternal recumbancy died within hours. On necropsy of one animal, the skin appears bruised and blackened due to extensive capillary rupture subcutaneously. The pericardial sac is filled with straw-colored fluid and there is excess fluid in both the thoracic and peritoneal cavities. The liver has grayish yellow, necrotic foci and flukes. What is the diagnosis? A - Tyzzer's disease B - Bacillary hemoglobinuria C - Clostridium hemolyticum D - Infectious necrotic hepatitis E - Blackleg

Infectious necrotic hepatitis (Black disease) is the result of both Clostridium novyi and liver flukes. When sheep eat spores of C. novyi on contaminated pasture soil, latent spores ultimately become lodged in the liver. The spores multiply in areas of liver necrosis caused by migration of liver flukes like Fasciola hepatica and Fascioloides magna. C. novyi produces a lethal necrotizing toxin that kills sheep and occasionally cows and pigs.

You serologically test 100 Siberian box turtles for galloping halitosis. 27 turtles test seropositive and 73 test seronegative. However, molecular testing reveals 3/27 of the seropositive turtles are disease free and 10/73 of the seronegative turtles are diseased. The entire fur-bearing turtle industry depends on your answer: What is the Predictive Value Positive (PVP) of your serologic test? A - 95% B - 89% C - 86% D - 73% E - 70%

It is 89%. Remember: you are comparing TWO TESTS here. PVP means "Of the turtles my test says are positive (27), how many are truly positive?" (27-3=24, this # goes in the "a" box) Here is how you do it: First, draw a 2x2 table, and label the boxes a,b,c,d. PVP = a/(a+b). Click here to see a Basic 2X2 table. Now, add in the TOTAL number of animals (100), the total positive by YOUR test (27) and the total negative by YOUR test (73), like this diagram: 2x2 with totals. Now the (slightly) tricky part. Add in the numbers that YOUR test got WRONG according to the gold standard test. (3 false pos in box b, 10 false neg in box c): Click here to see 2x2 with b and c cells. Last, subtract to fill in your "d" box (73-10=63) and do the math to calculate PVP = a/(a+b)=24/27=0.89 or 89% : Click here to see the final 2x2 with all cells filled and PVP calculated. FYI: You can calculate sensitivity a/(a+c), specificity d/(b+d), Predictive Value POS (PVP) a/(a+b) and Predictive Value NEG (PVN) d/(c+d) with the same 2x2 table.

Which one of the following drugs is used primarily to treat pituitary-dependent hyperadrenocorticism (Cushing's disease) in dogs? A - Fludrocortisone acetate B - Imidocarb C - Phenoxybenzamine D - Liothyronine E - Mitotane (o,p DDD)

Mitotane (o,p DDD) is one drug option used to treat pituitary-dependent hyperadrenocorticism (Cushing's disease). It selectively destroys the glucocorticoid-secreting cells of the adrenal cortex. REMEMBER dogs receiving mitotane should be given supplementary glucocorticoids during times of stress (ie: surgery, trauma, acute illness). Alternative treatments include l-Deprenyl (decreases pituitary ACTH secretion) or Ketoconazole (inhibits enzymes of cortisol synthesis). These are used in dogs who do not tolerate Mitotane. NOTE Trilostane is also used in treatment of hyperadrenocorticism. Survival times appear to be similar between Trilostane and Mitotane. Trilostane is typically given lifelong once or twice a day, while maintenance with Mitotane is given 2-3 times per week. Long-term, Trilostane is more expensive than Mitotane. In most respects, Trilostane appears to be safer but patients must be monitored for adrenal necrosis, which can lead to an Addisonian crisis and death if unrecognized. Experienced practitioners may disagree on whether to use Trilostane vs. Mitotane.

How many periods of estrus (heats) per year are typical in the dog? A - 2 B - 3 C - Depends on day length D - Depends on frequency of exposure to male E - 1

Most dogs have 2 heat cycles per year (range 1-3), except for basenjis which have one estrus per year, in the Spring. Dogs are UNseasonally monoestrus. The interestrus interval in dogs is generally 7 months (range 3 1/2 to 13 months). Click the following link for a table of reproductive cycle features, most species.

What does it mean if a cow is multiparous? A - Has given birth more than once B - Pregnant with twins C - Has been pregnant, then aborted D - Is a twin and other animal is male E - She is good at math

Multiparous means the dam has been pregnant and given birth two or more times. A female cow who has not yet had her first calf is called a heifer. Some farmers call them heifers until after the second calf is born, then they call them cows. On the farm you will often hear people refer to a "first calf heifer", meaning younger and not yet full size. Often if an animal is going to have trouble calving, it is a first calf heifer because she may be smaller than a full grown cow, or inexperienced at birthing. Expect dystocia in 10-15% of first-calf heifers. Parity in reproduction refers to the number of times a female has given birth. Primiparous means the first pregnancy and birth.

A newborn calf is presented with joints fixed in abnormal positions, kyphosis and a cleft palate. http://www.merckmanuals.com/vet/multimedia/v4738048.html Which exposure during pregnancy for the cow may have caused this problem in-utero in the calf? A - Lupine spp B - Bovine virus diarrhea infection C - Metaldehyde D - Gossypol E - Sorghum (Sudan grass, Johnson grass, Milo)

Newborn calves with arthrogryposis ("crooked calf") have ankylosed, rigid limbs, scoliosis, kyphosis, and sometimes a cleft palate. The most common toxic cause of arthrogryposis in the calf or lamb is consumption of toxic alkaloids (anagyrine) in Lupine spp plants by pregnant dams. Click here to see a Lupine plant. Adults that eat lupine may display inappetence, dyspnea, convulsions or death from respiratory paralysis. If lupines become infected with a fungus (Phomopsis leptostromiformis), mycotoxic lupinosis can cause hepatic damage. Infectious causes of congenital arthrogryposis include in-utero infection with Bluetongue virus or Akabane virus. Sorghum (Sudan grass, Johnson grass, Milo) can cause a neurologic toxicity, primarily in horses, or cyanide toxicity.

Which one requires a longer exposure time? A - Non-screen film B - Gridless film C - Screen film D - Film type does not matter

Non-screen film is primarily sensitive to x-rays, not light, requires LONG exposure times and produces radiographs with superb detail. Non-screen films are a good choice for intra-oral exams, dental studies, nasal radiographs and x-rays of bony extremities.

While conducting a routine physical on a 4 year old male intact bulldog, an irregular heart rhythm with a slow rate that is markedly slower on expiration is audible during auscultation. T=102.1 F (38.9 C)..[N=99.5-102.5 F] HR=60 bpm............[N=110-120] RR=24 brpm...........[N=15-34] What should be done next? A - Nothing B - ECG C - Echocardiogram D - Chest radiograph, CBC, blood chemistry panel E - Refer to for cardiology consult

Nothing. Extreme accentuation of sinus arrhythmia (bradycardia), markedly slower during expiration is a normal finding in brachycephalic breeds. No treatment needed if dog is not symptomatic. SYMPTOMATIC animals would present with fainting, weakness. If respond to an atropine test, consider Med Rx with Glycopyrrolate, Propantheline (Pro Banthine ®), Isoproterenol. If poor response, may need a pacemaker. Refs: Blackwell's 5-Min Vet Consult Canine Feline, 4th ed. pp.1262-3, Pasquini's, Tschauner's Guide to Small Animal Clinics, vol 1, 2nd ed. pp. 255-6 and the Merck Veterinary Manual online edition.

What is one cause of abdominal fat necrosis (lipomatosis) in cattle? A - Grazing tall fescue B - Chronic excess protein in ration C - Fatty liver disease D - Aflatoxicosis E - Pregnancy toxemia

One cause of abdominal fat necrosis (lipomatosis) in adult cattle (and some deer) is prolonged grazing of tall fescue infected with Acremonium coenophialum. Seen throughout the USA where tall fescue is the primary pasture grass. Over 90% of such pastures are infected with the endophyte. Even without fescue exposure, hard masses of necrotic fat are relatively common in adult cattle. On rectal exam, the masses feel like "floating corks" similar to cotyledons, and may be mistaken for a developing pregnant uterus. Remember the other fescue-related problem, fescue mycotoxins, which can cause lameness and hyperthermia in cattle and horses due to an ergot-like mold on tall fescue grass. Fatty liver disease in cattle is a complex metabolic imbalance that can occur when an overconditioned cow reduces feed intake. Most common in periparturient cattle (think of fat cows at calving).

Assuming the prevalence of heartworm in the area is 10%, what is the predictive value positive (PVP) of the test? A - 80% B - 84% C - 88% D - 92% E - 99%

PVP is 84%. The trick with this kind of question is to pick an imaginary number of animals that you test, like 1000, and fill out your 2x2 table from there. Follow the links to see diagrams step by step. If prev is 10% then there must be 100/1000 dogs with heartworm and 900 dogs that are disease-free. A 96% sensitive test will correctly call 96/100 positive (box "a"), and IN-correctly call 4/100 negative, (box "c": these are the false negs). If 100/1000 animals are infected, then 900/1000 are disease-free. Your 98% specific test will correctly call 882/900 disease-free (box "d": 0.98 X 900=882) and IN-correctly call 18/900 positive, (box "b": these are the false pos). Now your a,b,c,d boxes are all filled, it is easy to calculate PVP=a/(a+b)=96/(96+18)=84%

A local swine farm reports an acute outbreak of fever and anorexia in 20% of the herd. Affected pigs are reluctant to move, walk stiffly on their toes and lie separately rather than piling in groups.They shift weight from foot to foot when standing and squeal pitifully when handled. There is widespread erythema and purplish skin discoloration of the ears, snout, and abdomen of some affected animals. A typical affected animal looks like this: http://www.merckmanuals.com/vet/zk/multimedia/v4736078/n/x.html Which one of the following choices is the most appropriate treatment? A - Lincomycin in feed- whole herd B - Penicillin IM- acute and chronic cases C - Cull acute and chronic cases D - Penicillin IM-acute cases; Cull chronic cases E - Lincomycin IM-acute cases; Cull chronic cases

Penicillin is the treatment of choice for acute cases. CULL Chronic cases. These are ACUTE cases of erysipelas Lincomycin is more frequently used to treat respiratory disease and the polyarthritis associated with Mycoplasma hyosynoviae. Follow these links to see images of pigs with erysipelas skin lesions: Image 1-Hinds and Image 2- classic "diamond" and Image 3-Typical, but less pathognomonic.

The major active ingredient in most IV euthanasia solutions is: A - Phenobarbital B - Phenytoin C - Pentobarbital D - Thiopental E - Potassium chloride

Pentobarbital is one of the most commonly used drugs in IV euthanasia solutions. For the most complete reference on veterinary euthanasia, see the 2013 AVMA Guidelines on Euthanasia. The AVMA report states IV injection of a barbituric acid derivative is the PREFERRED METHOD for euthanasia of dogs, cats, other small animals, and horses. Potassium chloride can be used in conjunction with general anesthesia. Phenytoin and lidocaine are often added to pentobarbital-containing euthanasia solutions to increase cardiac depressant effects. Thiopental is an ultra-short acting thiobarbiturate used for anesthesia induction and for short procedures.

A 3 year-old domestic longhair female spayed cat is presented at your clinic with hypersalivation, vomiting, diarrhea, tremors, stumbling and a temperature of 105.2 F (40.7 C). Her pupils are normal size and responsive to light. While the vet examines the animal, she has a seizure. The owner relates that he had treated her the night before with a flea dip he had originally bought for his dog. What toxic ingredient do you suspect most highly? A - D-Limonene B - Carbamate C - Permethrin D - Organophosphate

Permethrin toxicity in cats is relatively common occurrence after being treated with a concentrated permethrin-containing flea product labeled for dogs. Rx with METHOCARBAMOL (Robaxin ®) a centrally-acting muscle relaxant and if needed, with seizure control (ie: diazepam, pentobarbitol, or inhalant anesthesia). UNLIKE OPP/carbamate toxicities, DO NOT Rx permethrin toxicity with Atropine. Big DDXs are Organophosphate (OPP) toxicity, and Carbamate toxicity which look similar. With OPP and Carbamates, may see MIOSIS, UNlike permethrin/pyrethroid toxicity. D-Limonene is a citrus-based anti-flea med that can cause vomiting in dogs. For a summary of toxicities, see the Zuku Review Top 20 Tox notes.

What does polychromasia mean in a red blood cell evaluation? A - Different sized cells B - Different colored cells C - Abnormally pale cells D - Irregularly-shaped cells

Red blood cells can be described by size, shape and color. Polychromasia means different colors or shades. Follow these links to see canine RBCs WITH polychromasia and feline RBCs WITHOUT polychromasia (both also show anisocytosis (variable sizes), due to response to an anemia). Poikilocytosis means irregularly-shaped cells. Follow this link to see poikilocytosis in a goat (which is common and normal in goats). Anisocytosis means different-sized cells. Follow this link to see anisocytosis in a cow. Microcytosis means small cells and is sometimes seen in dogs with a congenital liver problem called Portosystemic shunt (PSS).

Pyrrolizidine alkaloid toxicity is caused by chronic ingestion of which one of the following plants? A - Astragalus spp. (locoweed) B - Lupinus spp. (lupine) C - Nerium spp. (oleander) D - Persea spp. (avocado) E - Senecio spp. (ragwort)

Senecio spp. (ragwort). Common plants containing pyrrolizidine alkaloids (PA) are: Senecio vulgaris, S. jacobea, Amsinckia intermedius, Heliotropium europaeum, Crotolaria spectabilis. Although generally not palatable, livestock will eat these plants when baled in hay or on pasture when forage is scarce. Chronic ingestion allows accumulation of toxic levels of PA, resulting in hepatic fibrosis. Poisoning is most common in horses and cattle, sheep and goats are more resistant to toxic effects. Persea spp. (avocado leaves) contain persin. Signs of toxicity are: noninfectious mastitis, abrupt cessation of milkflow, heart failure. Nerium spp (oleander) contain cardiac glycosides. Signs of toxicity are: sudden death, weakness, diarrhea, cardiac arrhythmias. Lupinus spp. (lupine) contain alkaloids. Signs of toxicity are: birth defects (ingestion at 40-70 days in cattle), abortion, tremors, incoordination, head pressing, seizures. Astragalus and Oxytropis spp. (locoweed) contain alkaloids (swainsonine). Signs of toxicity are: excitability, incoordination, difficulty eating, exaggerated mouth movements, depression.

A 12-year-old neutered male mixed-breed cat presents with weight loss, polyphagia, polydipsia, polyuria, and unkempt haircoat. The cat is thin and has tachycardia. Which one of the following tests is most likely to confirm the presumptive diagnosis? A - Serum T4 B - Abdominal radiographs C - Urinalysis D - Abdominal ultrasonography E - Fecal examination

Serum T4. The signalment and clinical signs are highly compatible with hyperthyroidism. Canned food diet, ectoparasiticide exposure, and mixed breed origin are probable risk factors for Hyperthyroidism. The three treatment options are antithyroid medication, radioactive iodine therapy, and surgical thyroidectomy.

A 7-week old male Yorkshire terrier is presented with a 2 week history of on and off vomiting and diarrhea that began around the time he was weaned. The owners relate that he seems to "drink and pee a lot". They report pacing, disorientation, weakness, and "stumbling around". Physical exam is unremarkable, but only one testicle has descended. As the puppy explores the room he appears ataxic, stumbles a few times, and bumps his head into the wall. What is the clinical diagnosis? A - Congenital hiatal hernia B - Portosystemic shunt C - Canine distemper D - Diabetes insipidus E - Lead poisoning

Signs of hepatic encephalopathy (ataxia, disorientation, vomiting, diarrhea) beginning after weaning combined with polyuria/polydipsia ("drink and pee alot") in a cryptorchid male Yorkshire terrier says Congenital Portosystemic Shunt. Seen most in pure-breeds. Think SMALL Maltese, Yorkshire terrier, Min. Schnauzer. (But can see in Old English sheepdog, Irish Wolfhound) Usually in YOUNG animals, especially after weaning. 50% of males are CRYPTORCHID. Lead poisoning can cause vomiting, diarrhea and CNS signs (blind, hyperactive, seizures). Can see PU/PD in older animals but Hx here puts a shunt first on DDX.

A rapid ELISA test kit for psittacine beak and feather disease (PBFD) is being tested. Here are simulated test results, compared to a gold standard test for PBFD. ..........................PBFD pos....PBFD neg....Total ELISA positive.....111...........132............243 ELISA negative.....21.........1736..........1757 Total....................132.........1868.... ......2000 What is the specificity of the new ELISA test for PBFD? A - 1736/1868 B - 1736/1757 C - 111/132 D - 132/1868 E - 111/243

Specificity=1736/1868 d/(b+d) (93%) Start by drawing a 2x2 table, and label the boxes a,b,c,d. Specificity=d/(b+d) Click here to see a Basic 2X2 table. Psittacine beak and feather disease (PBFD) is caused by a psittacine circovirus. The name is misleading, because typical presentations do not have beak malformations and nowadays are less likely to show the severe feather abnormalities that were seen in Cacatua spp.(cockatoos) when PBFD first emerged. Click here to see a sulfur-crested cockatoo with PBFD. PCR screening has decreased prevalence of PFBD virus in cockatoos, though the disease can be seen in other old world psittacines as well, like Eclectus and African Gray parrots and lovebirds.

An 8 month old Rhodesian Ridgeback neutered male dog is presented with a complaint of a chronic concretion of hair, sebum, and discharge on the dorsal midline. Physical examination reveals no abnormalities other than the accumulation of debris on the dorsal midline with no local heat or swelling. There are no neurologic signs noted. The site is carefully clipped and prepared and a small, skin-lined cavity is discovered. Which one of the following is the most appropriate treatment? A - Oral administration of two doses of ivermectin 2 weeks apart B - Administration of broad spectrum antimicrobials with excellent bone penetration until resolved C - Closure of cavity with absorbable suture in a cruciate pattern D - Surgical excision of the dermoid sinus tract E - Careful cleaning of area and daily instillation of broad-spectrum antimicrobial cream until resolved

Surgical excision of the dermoid sinus tract. This is the clinical presentation of dermoid or pilonidal sinus. Dermoid sinuses (scroll to bottom of page) are congenital neural tube defects most frequently observed in Rhodesian Ridgeback dogs, in which they are believed to be hereditary. The sinus may be a blind-ended sac terminating just beneath the skin, or, in more serious cases, may communicate with the dura mater. In cases of infected sinuses communicating with the dura mater, meningitis with associated neurologic signs may result. Additional information about the extent of the sinus may be gathered via fistulography and myelography if necessary. Treatment involves complete surgical excision of the sinus tract. Click here to see a good summary on dermoid sinus or pilonidal sinus with images, courtesy of the American College of Veterinary Surgeons (ACVS).

Several sheep are sick at a petting-zoo that has cows, horses, pigs, bison and white-tailed deer. One of the deer is also affected. Affected sheep are listless and off-feed, with serous or mucopurulent nasal discharge and rectal temperatures ranging from 105-107.5°F (40.5-42°C). Physical exam shows swollen muzzles with edema and congestion of the lips, nose and face with small hemorrhages and ulcers on the mucous membranes. The ulcers appear where the teeth contact the swollen lips and tongue. Two affected sheep are lame. What is the diagnosis? A - Bluetongue B - Peste des petits ruminants (PPR) C - Caprine arthritis encephalitis D - Contagious ecthyma (soremouth) E - Foot-and-mouth disease (FMD)

Swollen sore muzzles with mucous membrane erosions, high fevers and lameness suggests infection with Bluetongue virus. Bluetongue is almost exclusively seen in sheep, though white-tailed deer, pronghorn antelope and desert bighorn sheep in North America can be severely affected. Rare in cows. REPORTABLE. Bluetongue is indistinguishable from Foot and Mouth disease (FMD), so that is a good second choice. But FMD is unlikely in the scenario above because FMD mainly affects pigs and cattle.

A 7 year old female spayed Border Collie is presented with two very goopy, gunk-covered eyes. A Schirmer tear test finds less than 10 mm/minute of wetting, a decreased result from the normal of at least 15mm / minute of wetting. The owner reports that the dog has been on "some kind of medicine" for the last 10 days, but it is actually his girlfriend's dog, and he doesn't know what the medicine is. Keratoconjunctivitis sicca (KCS) secondary to the drug is suspected. Which drug may be causing the KCS? A - Amitraz B - Trimethoprim sulfa C - Griseofulvin D - Itraconazole E - Prednisolone

Systemic sulfonamides like trimethoprim sulfa have been associated with keratoconjunctivitis sicca (KCS), sometimes irreversibly. Another drug-associated cause of TRANSIENT KCS is the combination of recent general anesthesia and atropine. Other causes of KCS include: Distemper, Immunologic (think ATOPY), Breed (Pugs, Yorkies), and trauma (Proptosed eyeball).

The menace response is used to test which cranial nerves? A - II and VII B - I and VIII C - V and VII D - IX and X E - V and VIII

The menace response is used to test visual input (cranial nerve II - optic) and blink response (cranial nerve VII - facial). Take care not to induce air movement which would elicit a tactile response (cranial nerve V - trigeminal) instead of visual.

An 8 year-old spayed female DSH cat is presented for right head tilt of 24 hours duration. The cat is up to date on vaccinations. Physical exam shows horizontal nystagmus with the fast phase to the left. The rest of the physical exam is unremarkable. What can the owner be told regarding the cat's treatment and prognosis? A - Fair prognosis with corticosteroids and antibiotics B - Good prognosis with corticosteroids and anti-emetics C - Excellent prognosis with corticosteroid therapy D - Excellent prognosis with corticosteroids and antibiotics E - Excellent prognosis without medication

The nystagmus usually improves markedly within 72 hrs and the cat will likely completely return to normal in the next 2-3 weeks without any medication. This cat most likely has idiopathic vestibular disease. For severe disorientation, consider sedatives (diazepam and acepromazine). Glucorticoids do not usually alter course of disease. Anti-emetics usually not effective. In cats where you cannot rule out otitis externa or media, antibiotic therapy is warranted.

What is the primary clinical presentation in chickens infected with Campylobacter jejuni? A - Enterocolitis B - Nothing C - Diphtheritic tracheitis D - Depression, anorexia, lethargy E - Hemorrhagic diathesis

The preferred answer is "nothing". Campylobacter jejuni from contaminated chicken is one of the leading causes of enterocolitis IN HUMANS, but the chickens themselves are asymptomatic. Click for more on avian campylobacter.

Which nerve block would most specifically relieve lameness resulting from fracture of the navicular bone? A - Palmar digital B - Abaxial sesamoidean C - Low four point D - Median and ulnar E - Tibial and peroneal

The palmar digital nerve block (also called a PD or heel block) would anesthetize the palmar third of the foot, including the navicular bone. All of the other nerve blocks can desensitize the navicular bone too, but would not be as specific as a palmar digital nerve block. The abaxial sesamoidean nerve block (ASNB, also called a pastern or foot block) would anesthetize the entire foot and much of the pastern. Sometimes an ASNB is needed to completely eliminate or diagnose navicular lameness, but it is less specific than the PD block. A low four point nerve block (Also called Low palmar or Volar block) would desensitize the fetlock and areas distal to it. The median and ulnar nerve block would anesthetize the carpus and areas distal to it. Click here to see forelimb nerve block landmarks. The tibial and peroneal nerve block would anesthetize the tarsus and areas distal to it. Click here to see pelvic limb nerve block landmarks.

A 7 year-old intact female poodle is presented. The owner has noted a lot of drinking and some urinary accidents in the house in the last week and that the dog hasn't seemed herself either. Yesterday she vomited twice. On physical exam the dog appears depressed and listless, has dark injected mucous membranes and a distended tender abdomen. T=104 F (40 C)..[N=99.5-102.5 F] HR=100 bpm.....[N=110-120] RR=30 brpm......[N=15-34] What diagnosis is of most immediate concern? A - Pyometra B - Intestinal foreign body C - Hepato-splenic tumor D - Diabetes mellitus E - Pregnancy

The preferred answer is canine pyometra. Pyometra is a life-threatening emergency which typically requires immediate surgery (ovariohysterectomy) if the cervix is closed and there is no obvious discharge. Open cervix pyometras are likely to present with a purulent discharge, simplifying Dx, but surgical ovariohysterectomy is still STRONGLY recommended. Pyometra should always be near the top of your DDX when presented with an older intact female dog or cat with a fever and distended abdomen. Follow these links to see a pyometra radiograph in a Norwegian Elkhound and to see a pyometra ultrasound in a cat.

What is the function of the proventriculus in the chicken? A - Glandular stomach B - Gizzard C - Replaces function of teeth D - Avian version of gall bladder E - Vocalization in males

The proventriculus is the glandular stomach in birds. The ventriculus (also called the gizzard), is the muscular stomach that contains grit and small rocks, and helps birds grind up hard seeds and other foods, in effect, acting like teeth. The crop is a diverticulum of the esophagus, used for temporary food storage that empties into the proventriculus. The purpose of the crop is to allow a bird to quickly swallow what food it can, then fly to safety and digest the meal at leisure. In hand-fed pediatric birds (usually valuable parrots), there are many potential problems possible with crop burn, crop puncture or proventricular impaction.

A bovine diet that is low in thiamine or high in sulfur can cause ____________. A - Polioencephalomalacia B - Pregnancy toxemia C - Parturient paresis D - Downer cows E - Pseudorabies

The two most common causes of polioencephalomalacia (PEM) are low thiamine (due to thiaminase activity from plants like bracken fern or low thiamine diets) and high sulfur in the diet (which can come from a high molasses-urea diet, corn or sugar cane byproducts, water, or other plants, including alfalfa, Canada thistle (Cirsium arvense), kochia, (Kocchia scoparia), and lambsquarter (Chenopodium spp).

During a routine dental cleaning under isoflurane anesthesia on an 8-year old male neutered Doberman, the ECG monitor shows the following pattern. The dog is stable and doing fine. What is this pattern? DOG ECG A - Atrial fibrillation B - Ventricular premature complexes C - Atrioventricular (AV) block D - Accelerated idioventricular rhythm E - Sinus arrhythmia

These are ventricular premature complexes (VPCs). You had better be thinking the dog is in early stages of Dilated Cardiomyopathy (DCM). According to Merck, 9th ed. "...ventricular premature contractions on a routine ECG in a presumed healthy DOBERMAN Pinscher or BOXER is HIGHLY SUGGESTIVE of CARDIOMYOPATHY". Echocardiography is the test of choice for definitive diagnosis of DCM. Remember this mnemonic for DCM breed predispositions: "DCM in a BOX" (ie: "D_obes, C_ockers, M_assive dogs (giant breeds), in a BOX_er" ALWAYS FATAL. Death usually in 6 to 24 months after Dx. WORSE Prognosis in DOBES, generally survive less than 6 months from Dx.

This 3-year-old horse presents for evaluation of this dry, horny, wart-like mass on the distal forelimb. This horse also has smaller wart-like lesions on its muzzle. The other young horses in the herd have similar lesions. Which one of the following is the most likely diagnosis? A - Papillomatosis B - Melanomatosis C - Dermatophilosis D - Pediculosis E - Trombiculosis

These are warts, or papillomas, which is related to infection with equine papilloma virus. Equine papillomatosis occurs most frequently in young horses and are most commonly found on the inner surfaces of the pinna, muzzle, distal limbs, and genitalia. They are contagious, and since they usually resolve spontaneously, treatment is rarely pursued.

During a necropsy on a 4-year old Beagle who died suddenly, a severe right ventricular hypertrophy is noted. image: http://www.merckmanuals.com/vet/zk/multimedia/v4730405/n/x.html Which condition is highest on the differential diagnosis list? A - Mitral dysplasia B - Tricuspid dysplasia C - Aortic stenosis D - Patent ductus arteriosus (PDA) E - Pulmonic stenosis

Think Pulmonic stenosis if you see RIGHT ventricular hypertrophy, because pulmonic valves blocks outflow from R ventricle (mostly dogs). Follow this link to see a Merck image of Rt. ventricular hypertrophy. Heard as a systolic murmur heard most loudly LEFT chest between the 2nd and 4th intercostal (IC) space; genetic link in beagles. Predilection seen in many breeds, including Eng. Bullldog, Min. Schnauzer, Scotties, Chihuahuas, Cockers, Boxers. Mitral stenosis can be confused with aortic stenosis (also called sub-aortic stenosis [SAS]), a systolic murmur which may be heard most loudly on the left chest between the 2nd and 5th intercostal (IC) space or at the thoracic inlet (lateral to trachea). Mitral dysplasia and other mitral valve problems are heard further back on left at 5th-6th IC. More common in CATS. Tricuspid dysplasia is heard further back on RIGHT at 5th-6th IC. Uncommon. Expect a continuous murmur with PDA. Vast majority detected at first vaccination visit.

A 12-year old male neutered cat weighing 14 pounds is presented with a 2-month history of PU/PD, increased appetite, lameness, weight gain, exercise intolerance and dyspnea. Physical exam shows a systolic heart murmur with a gallop rhythm, a lateral chest radiograph shows pulmonary effusion and a large heart. T=102.0 F..[N=100-103.1F] HR=110 bpm.......[N=130-140] RR=24 brpm........[N=16-40] A CBC shows PCV=48 %.............[N=24-45%], WBC=14,850..[N=3800-19,500] Neuts=88%............[N=35-75% ], Lymphs=4%..[N=20-55%] Monos=7%.............[N=1-4%], Eos=1%..[N=2-12%] Basos=rare............[N=rare] Blood chemistry reveals the following NA=150................[N=151-161], K=5.1..[N=3.5-5.1] LDH=200...............[N=35-225], ALT=108..[N=8.3-53] Total protein=10.1..[N=5.7-8.0], Glucose=350..[N=63-132] Alk Phos=200.........[N=3-65], Cholesterol=250..[N=95-130] BUN=47 mg/dL.......[N=10-30], Creatinine=3.4 mg/dL..[N=0.8-2.0 ] Bilirubin (total)=0.3.[N=0.0-0.2] Urinalysis U Sp. G= 1.018........[N=1.020-1.040] Glucose +++, WBC ++, RBCs +, protein +++ Which one of the following choices is the most likely diagnosis? A - Hyperthyroidism complicated by renal disease B - Acromegaly C - Pancreatic exocrine insufficiency D - Hyperadrenocorticism E - Diabetes insipidus complicated by cardiomyopathy

This complicated mix of diabetes mellitus , renal disease and heart failure/cardiomyopathy in an OLDER MALE cat suggests feline acromegaly. First presenting sign may be PU/PD, polyphagia of diabetes. WEIGHT GAIN in an unregulated diabetic cat STRONGLY SUGGESTS acromegaly, (but they may LOSE weight at first). Follow this link to see pulmonary edema and a large heart. May see prognathism (long mandible), lameness (esp. cats), marked vertebral spondylosis, thickened skin, large head, wide interdental spaces and a Stress leukogram (High neuts, low lymphs/Eos)

A 3 year old Standardbred mare is presented with a 2-month history of exercise intolerance. Endoscopy shows the following image. http://www.merckmanuals.com/vet/zk/multimedia/v4740629/n/x.html What is the diagnosis? A - Laryngeal hemiplegia B - Cleft palate C - Pharyngeal lymphoid hyperplasia (PLH) D - Dorsal displacement of soft palate E - Epiglottic entrapment

This is Dorsal displacement of the soft palate (DDSP). The caudal free margin of the soft palate moves dorsal to epiglottis, obstructing the airway and causing exercise intolerance. Rx conservatively, eliminating possible contributing diseases first (ie: rest, anti-inflammatories). Surgical treatments (Sternothyrohyoideus myectomy or soft palate resection) have mixed success rates around 50%. Epiglottic entrapment is a big DDX for DDSP. Outline of the epiglottis can still be seen with epiglottic entrapment, UNlike DDSP. Cleft palate is a newborn disease. See difficulty suckling, dysphagia, MILK DRIPPING from NOSTRILS. Click here to see a Cleft palate. Euthanize if severe. Surgical closure if small. Laryngeal hemiplegia ("Roarers") present with inspiratory noise during exercise and exercise intolerance. Click here to see laryngeal hemiplegia. More than 90% occur on LEFT side. Rx is surgery. Pharyngeal lymphoid hyperplasia (PLH) is common. Thought to be a normal immunologic event in younger horses.

A flock from a egg layer poultry operation is presented to investigate an outbreak of respiratory disease among a mixed-age population of adult chickens. The owner reports a sudden onset of sneezing, nasal discharge and facial swelling among about 30% of the birds. What is the clinical diagnosis? A - Infectious Bronchitis B - Fowl Cholera C - Infectious Coryza D - Infectious Laryngotracheitis E - Candidiasis

This is Infectious Coryza. Think acute respiratory disease with nasal discharge, sneezing, and SWELLING UNDER THE EYES. Caused by Avibacterium (Haemophilus) paragallinarum, infected flocks are a constant threat to uninfected flocks; farms with multiple-age flocks can perpetuate disease. In the USA, "All-in/all-out" management has essentially eliminated infectious coryza from many commercial poultry farms. Follow this link to see original Merck image of Infectious Coryza. Infectious Laryngotracheitis (ILT) look for gasping, coughing, blood stained beaks, blood occluding trachea on necropsy. A highly contagious herpesvirus infection, severe forms of ILT, can have 50% mortality. In most states ILT is REPORTABLE. Follow this link to see an image of ILT. Infectious Bronchitis is characterized by respiratory signs, decreased egg production and poor egg quality. Classically may see "wrinkled eggs" with inf Bronchitis. Pasteurella multocida causes Fowl Cholera. Think sudden onset septicemia. Signs vary greatly. In acute fowl cholera, dead birds are first indication of disease. May see fever, depression, anorexia, oral mucoid discharge, ruffled feathers, diarrhea, increased respiratory rate. Follow this link to see an image of Fowl Cholera. Another name for Candidiasis is thrush. Think Candida albicans, thickened mucosa, whitish, raised pseudomembranes in crop, mouth and esophagus. Follow this link to see an image of Candidiasis.

What is wrong with this picture? Click here to see image. http://www.merckmanuals.com/vet/zk/multimedia/v4739218/n/x.html A - Pelvic fracture B - Tail root avulsion C - Left acetabular fracture D - Megacolon E - Sacrococcygeal dysgenesis

This is Sacrococcygeal dysgenesis, an inherited trait in Manx cats. Look for dysgenesis, agenesis of sacrum or other spinal column abnormalities. May present asymptomatic or with palpable lumbosacral abnormalities and LMN hind limb signs. (Hopping or crouched gait). May see nonprogressive urinary or fecal incontinence, chronic constipation. Sequelae include recurrent urinary tract infections, megacolon. NO treatment.

A cow is presented with pruritus and crusts around the face, as shown in image 1. Image 2 shows the result of a skin scrape. 1: http://www.merckmanuals.com/vet/zk/multimedia/v4737445/n/x.html 2: http://www.merckmanuals.com/vet/zk/multimedia/v4737580/n/x.html What is the diagnosis? A - Psoroptic mange B - Trombiculidiasis C - Cutaneous onchocerciasis D - Chorioptic mange E - Sarcoptic mange

This is Sarcoptic mange. Sarcoptes NOT common in cattle (think pigs, dogs). Look for crusty pruritus starting on the head and neck. Follow this link to see a Merck image of canine sarcoptic mange. On skin scrape, look for short legs and long UNsegmented pedicles on sarcoptes. In CONTRAST to the long legs and short UNsegmented pedicles of chorioptes.

You are looking at a canine vaginal smear through a microscope. At what stage of the estrous cycle is this dog? http://www.merckmanuals.com/vet/zk/multimedia/v4737985/n/x.html A - Cannot tell from this slide B - Anestrus C - Estrus D - Diestrus

This is diestrus. DI-estrus canine vaginal smears will have more than 10% nucleated, round-edged parabasal and intermediate cells and the reappearance of some neutrophils. Follow this link to see a DI-estrus vaginal cytology image. In contrast, think CORN FLAKES with ESTRUS. See > 90% CORNIFIED superficial cells (angular, sharp edges like corn flakes, tiny pyknotic nuclei or no nuclei) when you think canine estrus, and NO NEUTS. Follow this link to see an estrus vaginal cytology image. Full cornification usually coincides with receptivity. Serum progesterone can predict the LH surge and help estimate ovulation. RULE OF THUMB: Breed bitch as soon as she will allow a male to mate or as soon as you see > 90% cornified superficial epithelial cells in a vaginal smear. Typically breed every 2-4 days until bitch enters diestrus for maximum fertility success

What feline condition may cause the lesion evident in this image? (image was the caudal aorta [bifurcation] with blood clot) A - Hyperthyroidism B - Hypertrophic cardiomyopathy C - Diabetes mellitus D - Subaortic stenosis E - Systemic hypertension

This is an aortic thromboembolism, secondary to feline hypertrophic cardiomyopathy (HCM) the most common heart disease of cats. Physical signs of aortic thromboembolism include hindlimb paresis or paralysis, cyanotic nail beds, toe pads and cold extremities, decreased or absent pulses, contracted and painful hindlimb muscles, relentless crying. Don't confuse this presentation with the nonpainful plantigrade stance of diabetic neuropathy in cats, an uncommon sequella of diabete mellitus. Often, cats with HCM are asymptomatic and the problem is discovered on physical exam, with auscultation of a systolic murmur, evident in 80% of cats with HCM. Physical signs in cats with clinical HCM may include tachypnea, dyspnea, anorexia, vomiting and lethargy

A 5-month old steer is presented that is unthrifty-looking and failing to gain weight. He appears somewhat lame, has a rough, light-colored hair coat and depigmented hair around his eyes like spectacles. The steer has a watery dark diarrhea full of gassy bubbles. What is the diagnosis? A - Selenium toxicity B - Copper deficiency C - Molybdenum deficiency D - Quercus (Oak) poisoning E - Magnesium deficiency

This is copper deficiency, which presents with "Ain't Doin' Right" ( ADR ) signs : ACHROMOTRICHIA (depigmented hair, especially around the eyes= "SPECTACLES") rough coat, decreased milk yield, lameness and decreased fertility, libido and "Peat Scours" also called "Teart" (severe scours with gas bubbles). Molybdenum toxicosis causes a secondary DEFICIENCY in Copper. Oak poisoning is more a gradual, multisystemic disease (nephrotoxic, hepatotoxic), can see PU/PD, hematuria, weight loss. Tetany is the classic presentation of Hypomagnesemia in cattle, along with hyperexcitability, ataxia, convulsions and death. Selenium toxicosis has many presentations depending on species, dose and length of time exposed.

An 8-year old male neutered domestic shorthair cat is presented with a one-month history of progressive exercise intolerance, panting, anorexia, weight loss and vomiting. On physical exam there is moderate cyanosis and a jugular pulse. On chest auscultation there is a systolic murmur loudest on the left side between the 5th and 6th intercostal space. T=102.1 F (38.9 C)..[N=100-103.1] HR=176 bpm...........[N=100-140] RR=40 brpm...........[N=16-40] A DV chest radiograph looks like the image below. What is the diagnosis? (shows valentine heart) A - Congestive heart failure B - Aortic insufficiency C - Hypertrophic cardiomyopathy D - Tricuspid insufficiency E - Taurine deficiency

This is the classic "Valentine heart" of feline Hypertrophic Cardiomyopathy (HCM).Note the very high RR and HR. Typically see HCM in cats 5-7 years old, more often in males. Cause unknown. Look for thickened L. ventricle wall on echocardiography. Follow this link to see thickened L ventricle on necropsy May see secondary pulmonary hypertension, edema, pleural effusion. DDX includes hyperthyroidism, systemic hypertension, acromegaly, congenital aortic stenosis. Systolic murmur loudest on L betw 5th-6th suggests MITRAL valve insufficiency (left AV). Aortic insufficiency is DIASTOLIC. Tricuspid valve murmurs (right AV) are loudest on the RIGHT (See Blackwell's, p. 905, Tschauner p. 212 for excellent murmur DDX tables). Taurine deficiency is associated with DILATED cardiomyopathy (DCM) in cats. Think more of LARGE BREED DOGS with DCM. Technically, HCM is a form of congestive heart failure, but the "Valentine" radiograph points you to the much more specific diagnosis of HCM.

A poultry operation has experienced sudden die-off of 5% of its' birds. Another 10% are sick with cyanosis and edema of the head, comb, and wattle. Many have subcutaneous ecchymotic hemorrhages on the shanks, feet and head. http://www.merckmanuals.com/vet/zk/multimedia/v4739972/n/x.html Some have a greenish diarrhea. On necropsy, petechial hemorrhages are visible on visceral organs and in muscles and there are blood-tinged oral and nasal discharges. A few birds that survived illness have developed torticollis, opisthotonos, and appear incoordinated. What test is most appropriate to confirm the presumptive diagnosis? A - Virus isolation from tracheal or cloacal swab B - Hemagglutination test on acute plasma C - Biopsy brain, spinal cord, proventriculus, gizzard D - Bacterial culture and isolation from visceral lesions on 5% blood agar E - Demonstration of toxin in serum, liver homogenates, or crop washings

This is the clinical picture of an outbreak of Highly Pathogenic Avian influenza (HPAI). AI viruses can be readily isolated from tracheal and cloacal swabs. AI viruses vary in pathogenicity. In general, see nothing with subclinical infections. If clinical signs appear, may see sinusitis and respiratory signs in low pathogenicity strains, and fulminating multisystemic, hemorrhagic signs with highly pathogenic strains. Think-edema, cyanosis of head, wattle, and comb; hemorrhagic discoloration of feet and legs, and petechial hemorrhages on visceral organs on necropsy. Follow these links to see leg hemorrhages on a chicken with AI. The acute form of Newcastle disease must be differentiated from HPAI by isolation of a hemagglutinating virus identified by inhibition with Newcastle disease antiserum.

A 14-month old female beagle-mix dog is presented with several firm non-painful swellings attached to her ribs. A thoracic radiograph shows the image below. http://www.merckmanuals.com/vet/zk/multimedia/v4739200/n/x.html What is the clinical diagnosis? A - Multiple cartilaginous exostoses B - Osteochondrosis C - Hypertrophic pulmonary osteopathy D - Chondrosarcoma E - Fibrosarcoma

This is the clinical picture of multiple cartilaginous exostosis (MCE-scroll down for details) also called osteochondroma, a benign proliferative disease of bone and cartilage typically seen in the metaphyseal region. Can be on any bone, including ribs, long bones, vertebrae (especially thoracic spinous processes). See in YOUNG dogs, cats, horses, humans. Stops when growth plates close. Often asymptomatic, but can see pain, lameness if compress overlying nerves, tendons. Can see neurological signs if compress spine. Follow this link to see the original Merck image with explanatory text. Do not confuse osteochondroma with OsteochonDROSIS (OC) or Osteochondrosis dissecans (OCD). Primarily a JOINT problem, (especially SHOULDER, stifle) in 4-10 month old rapidly-growing large breeds. Hypertrophic pulmonary osteopathy is a paraneoplastic disease middle-aged-older dogs/cats, secondary to mass in thorax, abdomen- see swollen distal limbs. Chondrosarcoma, Fibrosarcoma are older animal diseases typically > 6 years.

An otherwise healthy rabbit that keeps shaking his head and scratching incessantly. There is a brown crust coating the interior of both ears. What is the most likely diagnosis? A - Psoroptes cuniculi B - Cheyletiella C - Sarcoptes scabei D - Taenia serialis E - Otodectes cyanotis

This is the ear mite Psoroptes cuniculi, extremely common in rabbits worldwide. Click here for an image. The mites irritate the ear which causes serum and brown crusts to accumulate. Untreated infestations can cause secondary infections which can damage the inner ear. The crust can be cleaned with cotton soaked in dilute hydrogen peroxide, animal must be restrained or anesthetized for procedure. Mitacides approved for dogs and cats can be used, treat the inner ear plus the head and neck. SC ivermectin has been shown to be effective. Cheyletiella, also called walking dandruff, is a fur mite also common in rabbits. Sacroptes mange is rarely seen in rabbits, much more common in other animals, such as dogs and cattle. Taenia serialis is a rare larval worm infestion in rabbits.

A male gerbil is presented with a distinct oval area of alopecia on the mid-ventral abdomen. The animal has been observed rubbing this area on objects around its terrarium. What is the most appropriate interpretation? A - Normal, do nothing B - Common site of mammary gland adenocarcinoma, biopsy can confirm C - Sarcoptic mange, do a skin scrape D - Trichophyton mentagrophytes, do a fungal culture E - Demodex merioni infestation, treat with ivermectin

This is the normal location of the ventral marking gland (or pad) in gerbils. It is normal and more pronounced in males than females. Gerbils are often seen rubbing this area on things. This rubbing is thought to be a form of territorial marking. Demodex merioni occurs occasionally in older, debilitated gerbils.

This heifer has walked with an increasingly stiff and extended hock and stifle since 2 months of age. The gastrocnemius muscle is contracted, and the animal walks with short pendulum-like steps. http://www.merckmanuals.com/vet/zk/multimedia/v4738975/n/x.html What advice should be given to the owner? A - Cull B - Vitamin E supplements C - Radial neurectomy D - Quadriceps tenotomy E - Deep bedding, pectineus tenotomy

This is the typical presentation of spastic paresis, also known as Elso heel. Because spastic paresis is heritable, it is generally best to cull breeding animals. Look for an animal with a stiff hind leg in full extension, whether walking or standing. Affected animals (especially breeding bulls) should be eliminated. Palliative surgical treatments (gastrocnemius tenotomy, tibial neurectomy) are sometimes done to give relief and salvage the animal long enough to gain weight for slaughter.

A 9 year-old male neutered domestic shorthair cat is presented with a 4 week history of occasional disorientation and bumping into things. The cat does not visually track a ball rolled in front of him and has decreased pupillary light reflexes OU. On fundoscopic exam, the retinal vessels and disc are obscured by something in the way. What testing is indicated to assess the two most common underlying causes of this condition in cats? A - CSF analysis, CBC B - Thyroid function, BUN/Creatinine C - Adrenal function, Arterial pressure D - Hepatic function, Serum globulins E - Urinalysis, Pancreatic function, fasting blood glucose

Thyroid function, BUN/Creatinine. This presentation of vision loss in an older cat suggests retinal detachment, typically secondary to HYPERTENSION. Chronic renal disease may lead to HYPERtension and retinal detachment-Physical exam, history and BUN/creatinine will rule this disease in or out. HYPERthyroidism can ALSO cause HYPERtension. Your physical exam and history, plus a measure of T4 will help you evaluate this condition. Other diseases associated with retinal detachment include your alphabet diseases and 2 T's: FIP, FeLV, FIV Toxoplasmosis and Trauma

A cow is presented on emergency with urea/non-protein nitrogen toxicity. What is the treatment of choice? A - Rumenal infusion 2-8 liters vinegar,3-10 gallons cold water B - Relieve bloat, drench with 2-8 liters sodium bicarbonate C - Rumenotomy D - IV Fluids with MgSO4, Na thiosulfate PO E - Atropine, Protopam chloride IV q 4-6 hours

Treat Urea/non-protein nitrogen (NPN) toxicity with a rumenal infusion 2-8 liters 5% acetic acid (vinegar) and 3-10 gallons of cold water. The vinegar decreases rumenal pH which slows absorption of un-ionized ammonia. Repeat Q 6 hours up to 48 hours. Best results if animal is still ambulatory. It is often impossible to treat these cases before they die because of rapid progression to death. If possible, Rx with IV fluids. If necessary, relieve bloat. Urea/NPN toxicity is related to ammoniated feed toxicity which causes so-called "Bovine bonkers". Manage ammoniated feed toxicity by removing the ammoniated feed and treating severe cases with anticonvulsants like diazepam, pentobarbital.

A rabbit is presented with inflamed and chapped membranes of the anus and genital region. The genital area is scalded and raw, with brownish crusts and purulent exudate. What two conditions top the differential diagnosis list? A - Treponematosis, Hutch burn B - Tularemia, Cystitis C - Myxomatosis, Moist dermatitis D - Glomerulonephritis, Coccidiosis E - Pasteurellosis, Ulcerative pododermatitis

Treponematosis (rabbit syphillis, vent disease) and hutch burn (urine burn) resemble each other and are often confused. Treponematosis (rabbit syphillis, vent disease) is a venereal disease of rabbits caused by Treponema paraluis cuniculi. Affects the genitalia, may affect eyes and nose. Click here to see a rabbit with cutaneous treponematosis. Hutch burn is caused by wet and dirty floors, affecting the anus and genitalia. Click here to see a rabbit with hutch burn. Remember that cauda equina neuritis (polyneuritis equi) in horses may present with urine scald on the thighs. Other signs include a weak tail, hypotonic anus, urine dribbling and fecal retention. There may be a history of rubbing or chewing the tail head.

A 5-year-old Quarterhorse gelding is presented with a 4-day history of progressive asymmetric neurologic signs and muscle wasting. A Western immunoblot test of CSF is positive for Sarcocystis neurona. The owner wants to treat, but does not want to spend a lot of money What treatment plan is most appropriate? A - Trimethoprim sulfa and Pyrimethamine B - Doxycycline and DMSO C - Erythromycin and Rifampin D - Thiabendazole and Flunixin meglumine E - There is no effective treatment

Trimethoprim sulfa and Pyrimethamine. There are two treatment alternatives for Equine protozoal myeloencephalitis (EPM). The only FDA-approved treatments for EPM are oral paste formulations of ponazuril (costly: $1000 for 28 days) and nitazoxanide (manufacture was discontinued in February 2009). An alternative is long-term treatment (6 months) with a combination of antifolate antibiotics like sulfadiazine or sulfamethoxazole and pyrimethamine, a human antimalarial drug. The sulfonamide can be given with or without trimethoprim. Remember that pyrimethamine must be given at least one hour before or after hay is fed to the horse. Prolonged treatment with antifolate drugs may cause anemia, which can be and prevented with a diet high in green forage or supplements. Monitor with biweekly CBC.

Abortions due to brucellosis tend to occur at what stage of pregnancy? A - First trimester B - Second trimester C - Typically an early embryonic loss, prior to pregnancy detection D - At any time during pregnancy E - Last half of pregnancy

Typically, Brucellosis causes abortion in the LAST HALF of pregnancy, from the 5th month onwards. You might remember "Bruce is usually late". Expect STILLBORN calves. Cows only abort ONCE. Brucellosis is REPORTABLE. The vaccines used are the Brucella abortus strain 19 vaccine or the RB51 vaccine, given to heifer calves 4-12 months old, along with a USDA tattoo in the right ear. Remember: "The right ear is the RIGHT EAR". BVD and Trichomoniasis (occasionally) can cause early abortion in cows, but MOST things cause abortion late, like 3rd trimester. Listeriosis and Trueperella pyogenes can cause abortion at any time during pregnancy. Note the name changes - Trueperella pyogenes used to be called Arcanobacterium pyogenes and before that it was classified as Corynebacterium pyogenes.

Which is more severe, periodontitis or gingivitis? A - No difference in severity B - Depends on patient C - Periodontitis D - Gingivitis

Typically, peridontitis is a more chronic and severe problem. Periodontal disease is a general term that includes gingivitis (gum inflammation) and peridontitis (more chronic, severe damage from bacterial plaque that involves gingiva, periodontal ligament, alveolar bone, root cementum). Periodontal disease is caused by accumulation bacterial plaque at the gingival margin (gumline) due partly to lack of adequate oral hygiene.

Which teeth are x-rayed using the parallel radiograph technique? A - Mandibular premolars B - Maxillary incisors C - Mandibular incisors D - Maxillary premolars

Use the parallel radiograph technique for mandibular premolars and molars.

A 2-week old calf is presented that is dehydrated and almost emaciated from weight loss. She has a 1 week history of watery, foamy diarrhea and exhibits tenesmus during examination. The calf is the only sick one in a mixed group of calves that range from 3 weeks to 2 months of age What is the diagnosis? A - Colibacillosis B - Cryptosporidiosis C - Coccidiosis D - Ostertagiasis E - Coronavirus

Watery diarrhea in a SINGLE animal (1-4 weeks) in a group, plus emaciation suggests Cryptosporidiosis. Expect a more acute, lethal presentation of hemorrhagic diarrhea in 1-4 DAY old calves with Colibacillosis, and multiple cases with the viral diarrheas like Rotavirus - (5d-2 wks old, self limiting); Coronavirus, (4-30d). OLDER-animal diarrheas on this list include Coccidiosis ( Older than 21 days and in ALL ages, bloody diarrhea, tenesmus) Ostertagiasis (anorexia, poor growth, diarrhea in less than 2 year olds on pasture- a chronic disease).

A 3 year old male cat is positive for feline leukemia virus (FeLV) by both ELISA and IFA tests. A complete blood count (CBC) shows PCV=19%.................[N=24-45%] with polychromasia, reticulocytosis, anisocytosis WBC=3,600...............[N=3800-19,500] with neutropenia, lymphopenia Thrombocytes=300,000/microliter..[N=300,000-700,000] In addition to feline leukemia, what other infection is suspected in this cat? A - Mycoplasma haemofelis B - Hemobartonella bigemina C - Toxoplasma gondii D - Chlamydophila felis E - Cytauxzoon variabilis

When you see regenerative anemia (polychromasia, reticulocytosis, anisocytosis) in a FeLV-positive cat, suspect coinfection with Mycoplasma haemofelis (or Mycoplasma haemominutum). Typically, the anemia of feline leukemia virus (FeLV) alone is NON-regenerative. Mycoplasma haemofelis (formerly called Hemobartonella felis) causes feline infectious anemia, and is treated with tetracyclines. Click here to see regenerative anemia on a blood smear. In the SE USA, Cytauxzoon felis must be differentiated from Mycoplasma felis in cats with regenerative anemias. Think of Toxoplasma gondii (with neurologic and ocular manifestations) more in association with feline immunodeficiency virus (FIV).

A five year old neutered male dog is presented with a raised 2 cm alopecic dermal mass on the lateral right hock, present for several months. A fine needle aspirate and cytology reveals a monomorphic population of round cells with dark blue staining intracytoplasmic granules. Based on the presumptive diagnosis, what is the best choice for a treatment plan? A - Wide marginal excision B - Benign neglect C - Fluocinolone D - Doxycycline E - Itraconazole

Wide marginal excision. Think of mast cell tumors when you see round cells with intracytoplasmic granules on a fine-needle aspirate. Follow this link to see mast cell cytology. Wide marginal surgical excision offers the best chance for a cure of mast cell tumors especially if the cell type is well differentiated, if margins are clear and there is no evidence of spread to local lymph nodes. The location of the mass on the lateral hock is a challenge in this case, because it may not be possible to make wide and deep margins without skin grafts or healing by second intention. Benign neglect is a good treatment option for masses confirmed on cytology to be cutaneous histiocytoma (pleomorphic round cells with clear blue cytoplasm-no granules). Histiocytomas are benign and usually resolve on their own within three months. Doxycycline is an antibiotic often used to treat tick-borne diseases. In acute Ehrlichia canis infections, light blue intracytoplasmic inclusion bodies, morulae, may rarely be found within peripheral white blood cells. Itraconazole is an antifungal drug used treat infections such as Aspergillosis, Cryptococcus, blastomycosis, histoplasmosis and coccidioidomycosis. In these cases, cytology would demonstrate dark staining fungal elements (hyphae, sperules, etc) extracellularly, and these elements would be much larger than the cells themselves.

Under what conditions is a very sensitive test used? A - Rare disease, Early diagnosis improves prognosis B - Lethal disease, Highly prevalent disease C - Treatment does not affect prognosis, Non-infectious diseases D - Common disease, infectious diseases E - Highly prevalent disease, Treatment does not affect prognosis

You need a very sensitive test if: 1. Disease is rare (ie: BSE), or 2. Early Dx improves prognosis (ie: HIV in people), or 3. The disease is highly lethal or consequences of missing a case are severe. (ie: Rabies, Brucellosis, BSE, Screw-worm, FMD, EIA) Remember that a HIGHLY SENSITIVE test will have very FEW false negatives. That means if a test is highly sensitive, you can TRUST a NEGATIVE TEST. This sounds contradictory, but it makes more sense if you review this sensitivity diagram. Sensitivity=a/(a+c). "a" are true positives. "c" are false negatives. If sensitivity is HIGH then "c" (FALSE negs) must be small. Therefore, high sensitivity means you can really trust a NEGATIVE result to be correct.

A stress leukogram is characterized by: A - Neutropenia, lymphocytosis, eosinophilia B - Neutrophilia, lymphocytosis, eosinopenia C - Neutrophilia, lymphopenia, eosinophilia D - Neutropenia, lymphopenia, eosinopenia E - Neutrophilia, lymphopenia, eosinopenia

You would expect to see neutrophilia, lymphopenia and eosinopenia. A stress leukogram may be caused by either endogenous release of corticosteroids due to stress (like taking a big test), or hyperadrenocorticism (Cushing's disease) or exogenous medication with corticosteroids. Typically you see INCREASED neutrophils, and DECREASED lymphocytes and eosinophils. Effects on total WBC vary- expect leukocytosis in dogs, variable effects in cows. Monocyte effects also vary.

Which lesions are most commonly associated with Marek's disease? A - Edematous facial swelling with sinusitis B - Hemorrhagic skin lesions C - Nerve enlargement D - Wrinkled eggs E - Blood in the trachea

hink of NERVE ENLARGEMENT with Marek's disease; Also distortion of the pupil, enlargement of feather follicles ("skin leukosis"=condemnation of carcass). May see one leg forward, one leg back, a transient paralysis. Think Infectious Laryngotracheitis (ILT) if there is blood occluding trachea on necropsy. Remember wrinkled eggs go with Infectious Bronchitis. Think Infectious Coryza with sinusitis, swelling under eyes.

A flock from a turkey farm is presented with a mysterious illness. Several dead birds are noted, mostly younger. Sick turkeys are listless, with drooping wings, unkempt feathers, yellow droppings. Among the sick older birds emaciation is observed. Necropsy shows a yellowish green, caseous exudate in the ceca, cecal ulcerations and thickening of the cecal wall. A typical liver looks like the image below. What is the diagnosis? http://www.merckmanuals.com/vet/zk/multimedia/v12503756/n/x.html A - Histomoniasis B - Necrotic enteritis C - Avian spirochetosis D - Coronaviral enteritis of turkeys E - Hemorrhagic enteritis of turkeys

his is Histomoniasis. The combination of characteristic "Bulls-eye" lesions on liver and cecal changes are pathognomonic. Caused by protozoan Histomonas meleagridis, transmitted in eggs of cecal nematode Heterakis gallinarum. Expect a depression/diarrhea presentation. See sudden death picture with Necrotic enteritis caused by Clostridium perfringens. Follow this link to see the so-called "Turkish towel" intestinal pseudomembrane of Necrotic enteritis. Signs of Avian spirochetosis are highly variable, may be absent: see listlessness, shivering, increased thirst, green/yellow diarrhea with increased urates early on. Caused by a tick-borne Borrelia. Look for characteristic enlarged, mottled spleen with petechial hemorrhages, similar to Marble spleen disease of pheasants. See depression, bloody droppings, substantial mortality with Hemorrhagic enteritis of turkeys. Follow this link to see hemorrhagic intestines. Follow this link to see characteristic enlarged spleen. Expect diarrheal presentation with Coronaviral enteritis of turkeys but NOT the characteristic cecal/liver lesions described on necropsy above.

A flock from a broiler chicken operation is presented to investigate a disease outbreak characterized by caseous accumulations in the throat and weight loss. A typical bird looks like the image below. Two diseases are suspected: it is either candidiasis ("thrush") or another, similar disease. What is the other disease and how might the diagnosis be confirmed? http://www.merckmanuals.com/vet/zk/multimedia/v4740161/n/x.html A - Trichomoniasis, microscopic smear exam B - Fowl cholera, tracheal aspirate culture C - Infectious coryza, AGID D - Aspergillosis, fungal culture E - Necrotic enteritis, fecal flotation

his is Trichomoniasis , caused by Trichomonas gallinae, and diagnosed by microscopic smear exam of the caseous oral exudates. Look for trichomonads. More a problem in PIGEONS, but can cause disease in chickens. Remember Trichomonas foetus in cattle causes infertility (early embryonic death actually, 1st 2 months pregnancy). Infected bulls are mechanical carriers to cows. Candidiasis is a fungal disease that can look SIMILAR to trichomonas in chickens. Follow this link to see a Merck image of Candidiasis. Caused by Pasteurella multocida, Fowl Cholera causes sudden onset septicemia with VARIABLE signs. (Sudden death, anorexia, depression, mucoid beak discharge, ruffled feathers, diarrhea, increased RR. Aspergillosis presents as respiratory disease. See fungi on microscopic smear, may see granulomatous lumps in lungs. See sudden death with Necrotic enteritis, caused by Clostridium perfringens. Follow this link to see the so-called "Turkish towel" intestinal pseudomembrane of Necrotic enteritis.


संबंधित स्टडी सेट्स

PHIL 102 - Final Exam Study Guide

View Set

Chapter 15- California Ethics and Laws- A.D Banker

View Set

Environmental Health: Epidemiology

View Set

Bio 11: Quiz on Water Properties

View Set

Neurological: Stroke, Spinal Cord Injury Ch. 62, 63, 42

View Set